Let $f$ fail to be of bounded variation on $[0, 1]$. Show that there is a point $x_0 \in [0, 1]$ such that $f$ fails to be of bounded variation on each nondegenerate closed subinterval of $[0, 1]$ that contains $x_0$.
real analysis proof question
0
$\begingroup$
real-analysis
-
1There are quotas 50 questions/30 days and 6 questions/24 hours, so that you $c$an plan posting your questions a$c$$c$ordingly. (If you try to post more questions, stackexchange software will not allow you to do so.) For more details see [meta](http://meta.math.stackexchange.com/questions/4742/should-we-ask-for-question-quotas-like-those-that-have-been-available-for-the-bi/4770#4770). So you cannot post more than 6 questions a day. Perhaps you could spend a little more time with formatting your questions and showing your work instead of posting them in a quick succession. – 2012-11-23
1 Answers
3
You can do this with a proof by contradiction. Suppose this is not the case, so that around each point you can find a nondegenerate closed subinterval where $f$ has bounded variation. The interiors of all these intervals forms an open cover of compact $[0,1]$, so one can cover $[0,1]$ with finitely many such open intervals. Can you see why this implies that $f$ must be of bounded variation on all of $[0,1$]?
-
0If $f$ does not vary much on each finite interval, it cannot vary much on the finite union of such intervals. – 2012-11-23